Chapter 14: Questions & Answers

1 English legal system

Question 1

Describe the first instance jurisdiction of the civil courts and explain the system of appeals in civil cases.

(10 marks)

Question 2

Explain the rules and presumptions used by the courts in interpreting statutes.

(10 marks)

2 Contract law

Question 3

In relation to contract law distinguish between offers andinvitations to treat and explain why it is important to make such adistinction.

(Adapted from ACCA June 2004)

(10 marks)

Question 4

Hilary advertised a printing press in a specialist trade journalfor £15,000. Eleanor wrote to Hilary offering to buy it for £10,000.Hilary replied by return of post saying she would accept £13,000. Whenshe heard nothing further from Eleanor, Hilary wrote again saying shewould accept £10,000.

Hilary wrote to Amy offering for sale an office computer for£1,000. The morning that she received the letter Amy wrote to Hilaryagreeing to buy at the asking price. After she had posted the letter,but before it was delivered, Amy changed her mind and sent Hilary a faxasking her to ignore the letter when it arrived.

Required:
Advise Hilary as to whether binding contracts exist between herself and:

AEleanor

BAmy.

(Adapted from ACCA June 1997)

(10 marks)

Question 5

Explain the meaning of the following:

Athe principle that consideration must be sufficient but need not be adequate

Bthe principle that consideration may be executed or executory, but must not be past.

(10 marks)

Question 6

Raymond runs a small consultancy business, of which he is the soleproprietor, specialising in personal taxation advice. He entered into acontract with Samantha, a struggling artist, under which he agreed toprepare some draft business accounts for her, covering the last threeyears, for the sum of £800. Raymond completed the work but Samanthatold him she could only afford to pay £200 for the work. After a bitterargument Raymond reluctantly accepted a cheque for £200 from Samantha,which was stated to be in full and final settlement of the debt.

Shortly afterwards Samantha's paintings began to realise very highprices and Raymond has just read in a newspaper that her latest work hasbeen sold for £20,000. He now wishes to claim the balance of £600from Samantha and approaches you for advice.

Advise Raymond.

(Adapted from ACCA June 1996)

(10 marks)

Question 7

Grace, an accountant, works as a sole practitioner. She does notemploy any staff. Grace has recently won a lucrative contract fromExpansion Ltd to undertake all the company's payroll work. The initialperiod is for six months, renewable thereafter on a 12-monthly basis aslong as the work done by Grace is satisfactory.

In order to undertake the work for Expansion Ltd, Grace hires apowerful desktop computer together with dedicated software from OfficeSupplies Ltd. Grace signed a written hire contract with Office SuppliesLtd but she did not read it. It contained a clause stating that 'OfficeSupplies Ltd are not liable for any financial losses or other losses,however caused, occasioned by using hardware or software productssupplied by the company'.

Neither the computer nor the software operated correctly because ofnegligent design and manufacture by the company and as a result Gracefailed to make proper National Insurance deductions for the employees ofExpansion Ltd. Expansion Ltd consequently refused to renew the contractwith Grace and she suffered considerable loss in her profits as aresult.

Office Supplies Ltd admits that it has been negligent but deniesany liability to Grace on the basis of the exclusion clause in thecontract.

Advise Grace.

(Adapted from ACCA Dec 1994)

(10 marks)

Question 8 - Fixed Test 1

Describe briefly the main rules that govern:

Athe incorporation of exemption clauses into contracts

Bthe interpretation of exemption clauses in contracts.

(10 marks)

Question 9

State and explain the remedies available for breach of contract.

(Adapted from ACCA Dec 2004)

(10 marks)

3 The law of torts

Question 10

Albert wishes to rent a flat. He views 116 High Street and appliesto be a tenant. He fills out a form giving his accountant's details andthe landlord, Brad, applies to the accountant for a reference relatingto Albert's financial situation. The accountant, Charles, confusesAlbert with another client and does not bother to check the fileproperly. He replies to Brad that Albert is an excellent client withhigh income and capital and recommends him as a tenant.

In fact had he looked at the correct file, he would have seen thatAlbert was continuously in overdraft and had been in serious debt withvarious banks on and off for several years. The reference was suppliedwith the words: 'This firm will not accept any liability for inaccuracycontained in this reference or any loss incurred as a consequence'.

Some months later, Albert disappears having taken the flat on and run up several thousand pounds' worth of rent arrears.

Required:

Advise Brad as to whether he can recover his losses from Charles.

(10 marks)

Question 11

Mary is made redundant after working seventeen years as a nursewith Barnett Hospital. She receives a large sum of money by way ofredundancy payment.

Soon afterwards she is offered, subject to satisfactory references,a highly paid position with Comfort Home. The job offer is subsequentlywithdrawn and, when Mary makes further enquiries, she is given a copyof a reference supplied by Barnett Hospital which criticises her lack ofcommitment. When Mary confronts the hospital with the reference, theyapologise profusely and point out that they had confused her withanother employee with the same surname who was recently sacked forincompetence. Barnett Hospital informs Comfort Home and supplies a newreference but, by then, it is too late as Comfort Home have filled thevacancy with the second choice candidate.

Mary decides to invest her redundancy money into two companies.Shares in her ex-primary school, which is now offering privateeducation, are purchased on the advice of Joe an investment advisor andold friend that Mary met for a drink. The share price falls dramaticallyafter allegations that directors of the company are under investigationfor fraud and bribery.

Mary also purchase shares in MGS.com, a newly floated company onthe alternative investments market. The shares had been recommended inan article which Mary had downloaded from the Internet. The article hadbeen written by Thomas, an investment guru, well known for being aregular guest on television chat shows and for having made severalmillion pounds out of investments in internet companies. The articlefails to mention the highly speculative nature of investment in suchcompanies and the fact that, in most cases, what is being purchased isan idea rather than a proven record of performance. Six months later,Mary finds out the MGS.com shares are worthless.

Required:

Advise Mary whether she can recover her losses from:

ABarnett Hospital

BJoe

CThomas.

(10 marks)

4 Employment law

Question 12

Carol ran a business in Glasgow which specialised in producingcomputer software programmes. Dan and Eve both worked for Carol for aperiod of three years. They were both described as self-employed andboth paid tax as self-employed persons. Carol provided all of theirspecialist computer equipment. Dan was required to work solely on theprojects Carol provided, and he had to attend at her premises every dayfrom 9am till 5pm. Eve usually worked at home and was allowed to work onother projects. Eve could even arrange for her work for Carol to bedone by someone else if she was too busy to do it personally. When Carollost her most important contract, which Dan had been working on, shedecided to relocate the business to be closer to her one remaining largecontract in London some 500 miles away. As a result she told Dan andEve that there would be no more work for them.

Required:

Explain to Dan and Eve:

Ahow the courts decide whether someone is self-employed or is an employee

Binto which category they are likely to fall.

(Adapted from ACCA June 2000)

(10 marks)

Question 13

In July 20X5, Norman applied for, and obtained, the post ofmarketing consultant with Allpass Ltd, based at their Manchester office.In August 20X7, Allpass Ltd told Norman that, because of a reduction ofwork in Manchester, from November 20X7 he was to be transferred to itsBristol office, some 200 miles away from Manchester. Norman refused toaccept the transfer, arguing that he was employed to work in Manchester,and on 1 October 20X7 Norman wrote to Allpass Ltd, terminating hiscontract with the company.

Norman now wishes to pursue an unfair dismissal claim against Allpass Ltd.

Required:

Advise Norman on the following matters:

Athe significance of the fact that Norman terminated his contract with Allpass Ltd

Bhis likely chances of success in a claim for unfair dismissal.

(Adapted from ACCA Dec 1997)

(10 marks)

5 Agency law

Question 14

Explain the difference between agency of necessity and agency by ratification.

(10 marks)

6 Partnerships

Question 15

Clare, Dan and Eve formed a partnership 10 years ago. Two years agothe partnership employed Frank as its manager and last year Dan retiredfrom the partnership. Eve and Clare subsequently left much of theday-to-day work to Frank who has let it be known generally that he hasbecome a partner, although he has not. In January of this year Frankentered into a large contract with a longstanding customer, Greg, whohad dealt with the partnership for some five years.

Greg believed Frank's claim that he was a partner in the business.

This contract has gone badly wrong leaving the partnership stillowing £50,000 to Greg and unfortunately the business assets will onlycover the first £25,000 of the total debt.

Required:

Consider and explain the potential liabilities of Dan, Clare, Eve and Frank.

(10 marks)

Question 16

Detail the grounds upon which a partnership can be terminated.

(10 marks)

7 Corporations and legal personality

Question 17

Explain what legal limitations there are on the names that may beadopted by companies, paying particular regard to the tort of 'passingoff'.

(10 marks)

Question 18

Explain the meaning and effect of a company's articles of association, paying particular attention to the following issues:

AThe operation of the model articles of association.

(2 marks)

BThe effect of the articles on both members and non-members.

(4 marks)

CThe procedure for altering the articles of association.

(4 marks)

(Total: 10 marks)

Question 19 - Fixed Test 2

Fran, Gail, Hannah and Ian have just been made redundant and havereceived payments of £50,000 each. They were highly skilled workers andbelieve that they could carry on their own business successfully.

Required:

Explain four advantages of the registered company over thepartnership as a form of business organisation and advise them as towhich form of business best suits their situation.

You are not required to consider the Limited Liability Partnership.

(10 marks)

8 Capital and financing

Question 20

The board of Wealthy plc is considering lending a substantial sumof money to Hardup plc. As part of the consideration for the loan,Hardup plc has offered a debenture containing a floating charge over theassets and undertaking of the company. To help the board of Wealthy plcreach a final decision regarding the loan, you have been asked toprovide certain information concerning the transaction.

Required:

AExplain what is meant by a 'floating charge'.

BExplain whether the floating charge will provide adequate security for Wealthy plc.

CWhat initial steps should Wealthy plctake to ensure that the assets and undertaking of Hardup plc are notalready the subject of a floating charge and how does it ensure that itsown charge is a valid security?

(10 marks)

9 Directors

Question 21

Julie was appointed director of ABC Ltd in 20X5. Her servicecontract provided that she should hold office for five years and thisterm was also stated in the articles of association of ABC Ltd.

The other directors have now decided that Julie should be removedfrom her directorship. They tabled a resolution that Julie be removedfrom office and it was duly passed. Julie attended the meeting and made astatement that she intended to take legal advice as she believed shecould not be removed in breach of the articles of association and of herservice contract. The directors of ABC Ltd have asked for your advice.

Required:

Advise the directors, explaining whether the shareholders had theauthority to pass the resolution and suggesting what legal redress Juliemight have.

(10 marks)

Question 22

Len is a director of Mod plc, but he also owns a majority interest in Nim Ltd.

Last year Mod plc entered into a contract to buy new machinery fromNim Ltd. Len attended the board meeting that approved the contract andvoted in favour of it, without revealing any link with Nim Ltd.

At the same meeting the board of Mod plc decided not to pursue thedevelopment of a new product that had been offered to them by itsinventor. Len, however, liked the new product and arranged for it to beproduced by Nim Ltd. It has proved to be a great success and Nim Ltd hasmade a great deal of money from its production.

Required:

Owen is a shareholder in Mod plc and has found out about Len'slinks with Nim Ltd. He seeks your advice on whether any action can betaken against Len in relation to either:

Athe purchase of the machinery from Nim Ltd, or

Bthe development of the new product by Nim Ltd.

(Adapted from ACCA June 2000 examination)

(10 marks)

Question 23 - Fixed Test 3

GLM plc's articles of association contain the following regulation:

'No single director is empowered to enter into contracts on behalfof the company in excess of £250,000 without written authorisation bythe chairman of the board of directors.'

Matthew, the managing director, has recently committed the companyto a contract with PQR Ltd for the supply of equipment valued at£300,000. PQR Ltd demanded a copy of GLM plc's articles and askedMatthew whether he had obtained the consent of the chairman. Matthewstated that he had, which was untrue.

Required:

Explain whether the board of GLM plc can avoid the contract.

(10 marks)

10 Corporate administration

Question 24

AWhat are the statutory requirements for companies to hold annual general meetings?

(3 marks)

BWhat is a written resolution? When can it be used by a private company?

(7 marks)

(Total: 10 marks)

11 Insolvency

Question 25

In relation to company law explain:

Athe meaning of winding up

(3 marks)

Bthe procedures involved in:

Ia members' voluntary winding up

(3 marks)

II a creditors' voluntary winding up.

(4 marks)

(Total: 10 marks)

12 Corporate governance

Question 26

AWho are non-executive directors? Explain whether they play a useful role in company boards.

(5 marks)

BWhat guidance does the Higgs review give on the independence of NEDs?

(2 marks)

COutline the advantages and disadvantages of the system of self regulation introduced by the Combined Code.

(3 marks)

(Total: 10 marks)

13 Fraudulent behaviour

Question 27

Susan is a secretary working in her firm's corporate department.She overhears a senior lawyer in her firm taking a telephone call fromthe chair of AIM plc during the course of which she learns that atake-over bid for the share capital of AIM plc is to be announced by HITplc the following week. She tells her father, Joseph, that eveningabout what she heard. Joseph is a stockbroker and the next day he buysup traded option on the market to acquire AIM shares for £1 each. OnMonday the following week, HIT plc makes a public bid for AIM plc andAIM shares rises to £5 each. Joseph exercises his options over AIM plcshares and immediately resells them for £5 each, netting a profit of£4 per share.

Required:

Discuss whether Susan and Joseph have committed insider dealing.

(10 marks)

Question 28

Dome plc is listed on the stock exchange. Ewan works for Dome plcas an accountant. Whilst drawing up the annual accounts, Ewan noticesthat Dome plc's profits are better than anyone could have expected. As aconsequence of this knowledge, he buys shares in Dome plc before thegood results are announced. He makes a substantial profit on the sharedealing. Ewan also tells his friend Frank about the results before theyare announced. Frank also buys shares in Dome plc.

Required:

Have either Ewan or Frank done anything illegal and, if so, how is their action regulated?

(10 marks)

Test your understanding answers

Question 1

Civil courts

County court

County courts deal with the majority of the country's civillitigation. Jurisdiction includes contract and tort claims andundefended matrimonial cases.

The small claims track is for cases where the amount involved doesnot exceed £5,000, or if the parties agree to the procedure. Theprocedure is cheaper, quicker and more informal than a County courthearing.

The fast track is for cases under £15,000. Complex cases, and anycases over £15,000, are dealt with under the multitrack system.

High Court

The High Court has three divisions. The Chancery Division firstinstance jurisdiction covers bankruptcy, company matters, landlord andtenant, mortgages, probate, patents and copyright.

The Family Division first instance jurisdiction covers the wholerange of family matters including validity of marriages, divorce,legitimacy, adoption, guardianship, wardship and disputes concerning thematrimonial home.

The first instance business of the Queen's Bench Division is widebut mainly covers contract and tort actions. Jurisdiction overcommercial matters is exercised by a Commercial Court. The Division alsohas an Admiralty Court which deals with claims for damage, loss oflife, or personal injury arising out of collisions at sea, claims forloss or damage to goods carried in a ship, and disputes concerning theownership or possession of ships.

Appeal

Appeal from the County court and High Court generally lies to theCourt of Appeal (Civil Division) with a further and final appeal withleave to the House of Lords.

Exceptionally, appeal from the matrimonial and insolvencyjurisdictions of the County court lies to the Divisional Courts ofFamily and Chancery respectively with a further and final appeal withleave to the House of Lords.

Exceptionally appeal may be made with leave direct from the firstinstance jurisdiction of the High Court to the House of Lords. This'leapfrog' appeal is rare. It is only permitted on a point of law ofgeneral importance on which there is a pre-existing binding Court ofAppeal precedent such that the result of an appeal to the Court ofAppeal is a foregone conclusion and therefore a needless expenditure oftime and money, or where the case involves a point of statutoryinterpretation.

Question 2

Rules and presumptions

Judges use a number of rules and presumptions to help them decide the meaning of the words of a statute.

There are three main rules of statutory interpretation.

The literal rule

Under this rule, the judge is required to consider what thelegislation actually says rather than considering what it might mean. Inorder to achieve this end, the judge should give the words in thelegislation their literal meaning; that is their plain, ordinaryeveryday meaning, even if the effect of this is to produce what might beconsidered an otherwise unjust or undesirable outcome. In Fisher v Bell (1961)the court chose to follow the contract law literal interpretation ofthe meaning of the word 'offer' in the Act in question and declined toconsider the usual non-legal interpretation of the word.

The golden rule

This rule is applied in circumstances where the application of theliteral rule is likely to result in what appears to the court to be anobviously absurd result. It should be emphasised, however, that thecourt is not at liberty to ignore, or replace, legislative provisionssimply on the basis that it considers them absurd. It must find genuinedifficulties before it declines to use the literal rule in favour of thegolden rule. As examples, there may be two apparently contradictorymeanings of a particular word used in the statute, or the provision maysimply be ambiguous in its effect. In such situations, the golden ruleoperates to ensure that preference is given to the meaning that does notresult in the provision being an absurdity. Thus in Adler v George (1964),the defendant was found guilty, under the Official Secrets Act 1920,with obstruction 'in the vicinity' of a prohibited area, although shehad actually carried out the obstruction 'inside' the area.

The mischief rule

This rule permits the court to go behind the actual wording of astatute in order to consider the problem that the statute is supposed toremedy.

In its traditional expression, the mischief rule is limited bybeing restricted to using previous common law rules in order to decidethe operation of contemporary legislation. Thus in Heydon's case (1584) it was stated that in making use of the mischief rule, the court should consider the following four things:

IWhat was the common law before the passing of the statute?

II What was the mischief which the common law did not adequately deal with?

IIIWhat remedy for that mischief had Parliament intended to provide?

IVWhat was the reason for Parliament adopting that remedy?

Use of the mischief rule may be seen in Corkery v Carpenter (1950), in which a man was found guilty of being drunk in charge of a carriage although he was in fact only in charge of a bicycle.

In addition to the rules of interpretation, the courts may alsomake use of certain presumptions. As with all presumptions they arerebuttable and may be expressly overridden by the clear expression ofsuch an intention in the statute under consideration. The presumptionsoperate:

Iagainst the alteration of the common law

II in favour of the requirement that mens rea be a constituent in any criminal offence

IIIagainst retrospective application

IVagainst the deprivation of an individual's liberty, property or rights

V against application to the Crown

VIagainst breaking international law

VII   in favour of words taking their meaning from the context inwhich they are used. This general presumption appears as three distinctsub-rules. The noscitur a sociis rule is applied where statutoryprovisions include a list of examples of what is covered by thelegislation. It is presumed that the words used have a related meaningand are to be interpreted in relation to each other. The eiusdem generisrule applies in situations where general words are appended to the endof a list of specific examples. The presumption is that the generalwords have to be interpreted in line with the prior restrictive examples(Powell v Kempton Park Racecourse Co (1899)). The expressiounius exclusio alterius rule simply means that where a statute seeks toestablish a list of what is covered by its provisions, then anything notexpressly included in that list is specifically excluded.

The above should be read within the context of HRA 1998, whichrequires all legislation to be construed in such a way as, if at allpossible, to bring it within the ambit of the European Convention onHuman Rights.

Question 3

Offer

An offer sets out the terms upon which an individual is willing toenter into a binding contractual relationship with another person. It isa promise to be bound on particular terms, which is capable ofacceptance.

Offers, once accepted, may be legally enforced but not allstatements will amount to an offer. It is important, therefore, to beable to distinguish what the law will treat as an offer from otherstatements which will not form the basis of an enforceable contract.

An offer must be capable of acceptance. It must therefore not be too vague. In Carlill v Carbolic Smoke Ball Co (1893)it was held that an offer could be made to the whole world and could beaccepted and made binding through the conduct of the offeree.

Invitation to treat

However, most difficulties arise in relation to invitations totreat. These are distinct from offers in that they are invitations toothers to make offers. The person to whom the invitation to treat ismade becomes the actual offeror, and the maker of the invitation becomesthe offeree. An essential consequence of this distinction is that theperson extending the invitation to treat is not bound to accept anyoffers subsequently made to them.

The following are examples of common situations involving invitations to treat:

I     The display of goods in a shop window. In Fisher v Bell (1960)a shopkeeper was prosecuted for offering offensive weapons for sale, byhaving flick-knives on display in his window. It was held that theshopkeeper was not guilty as the display in the shop window was not anoffer for sale but only an invitation to treat.

II    The display of goods on the shelf of a self-service shop.In Pharmaceutical Society of Great Britain v Boots Cash Chemists (Southern) (1953),the defendants were charged with breaking a law which provided thatcertain drugs could only be sold under the supervision of a qualifiedpharmacist. They had placed the drugs on open display in theirself-service store and, although a qualified person was stationed at thecash desk, it was alleged that the contract of sale had been formedwhen the customer removed the goods from the shelf. It was held thatBoots were not guilty. The display of goods on the shelf was only aninvitation to treat. In law, the customer offered to buy the goods atthe cash desk where the pharmacist was stationed.

III    A public advertisement. In Partridge v Crittenden (1968)a person was charged with 'offering' a wild bird for sale contrary tothe Protection of Birds Act 1954, after he had placed an advertisementrelating to the sale of such birds in a magazine. It was held that hecould not be guilty of offering the bird for sale as the advertisementamounted to no more than an invitation to treat.

This should be contrasted with Carlill v Carbolic Smoke Ball Co (1893),where the relevant newspaper advertisement was held to be an offer. Thedifference is due to the intention of the advertiser and the fact thatthe contract envisaged in Carlill is a unilateral contract andnot a bilateral contract. In other situations where a unilateralcontract is envisaged (the 'reward' cases), advertisements are treatedas offers.

IV   Tenders. A tender is regarded as the offer which the person or body seeking the goods or services is free to accept or not.

V    A share prospectus. Such a document is merely an invitation to treat, inviting people to make offers to subscribe for shares in a company.

Importance of the distinction

The distinction is important in order to decide when the contractcomes into existence. If the statement is merely an invitation to treat,both an offer and an acceptance are required in order to bring thecontract into existence. However, if the statement is an offer, only anacceptance is required in order to bring the contract into existence.

Question 4

AEleanor

An offer is a definite and unequivocal statement of willingnessto be bound by contract on specified terms without further negotiations.An offer can be contrasted with an invitation to treat, which is aninvitation to the other party to make an offer.

Generally, advertisements are invitations to treat, not offers – Partridge v Crittenden (1968). Exceptionally, advertisements are offers where no further negotiations are intended – Carlill v Carbolic Smoke Ball Co (1893).

Here, Hilary's advertisement is an invitation to treat. Eleanor's reply is an offer.

Acceptance is the absolute (unconditional) and unqualified assentto all the terms of the offer. A counter-offer is an offer made inresponse to an offer – Hyde v Wrench (1840).

Hilary's reply by return of post is not an acceptance: it is acounteroffer because it attempts to negotiate the price up to £13,000. Acounter-offer terminates the original offer; therefore Eleanor's£10,000 offer no longer exists. Accordingly, when Hilary writes againsaying she would accept £10,000, this is not an acceptance, it is afresh offer which Eleanor then rejects.

In conclusion Hilary is advised that no binding contract exists with Eleanor.

BAmy

Hilary's initial letter is an offer. Amy's letter replying is anacceptance. Once an acceptance has been made a contract comes intobeing. Therefore it is not possible to 'revoke' an acceptance: any suchattempt is a breach of contract.

In general, acceptance is effective once it has been communicated to (i.e. received by) the offeror – Entores v Miles Far East Corporation (1955). However, under the postal rule, a posted acceptance is effective when the letter is posted, not when it is received.

The postal rule only applies provided:

(1)the letter is properly stamped and addressed, and

(2)it is put into the hands of the postal authorities, and

(3)it is reasonable to use the post asa means of communication, in the sense that the post would be withinthe contemplation of the parties.

If the postal rule applies, acceptance will be effective when theletter is posted. Assuming the letter was properly stamped andaddressed, the only issue is whether it was reasonable to use the post.As Hilary used the post for her offer it is strongly arguable that itwas reasonable for Amy to reply by post.

In conclusion, a contract existed between Hilary and Amy from themoment Amy posted her acceptance. Her later fax cannot revoke it.

Question 5

Consideration principles

AIn considering the validity of acontract, the courts are generally unconcerned as to whether the partiesto the contract have made a good or a bad bargain – that is regardedas a matter for the parties themselves and is not an issue on which thecourts will interfere.

However, although consideration need not be adequate, it must besufficient – in other words, it must have some value in the eyes ofthe law. There are a number of situations where consideration has beenheld to be insufficient:

I     The case of Collins v Godefroy (1831) established that the performance of an existing statutory duty is not sufficient consideration to establish a contract.

II The performance an existing contractual duty is also not sufficient: Stilk v Myrick (1809).

III   By contrast, however, performance beyond that already legallyrequired is capable of amounting to sufficient consideration: Glasbrook Bros Ltd v Glamorgan County Council (1925).

BOne of the well-established principlesof the law of contract is that while consideration may be executed orexecutory, it must not be past. Executory consideration in effectconsists of a promise in return for a promise. Executed consideration isa promise in return for an act. Thus, if X promises to dig Y's garden,in return for Y's promise to pay X £50, there is a valid contractbetween them, with both parties having obligations to perform in thefuture. The consideration is executory. Once X has dug Y's garden, he issaid to have provided executed consideration for Y's promise to pay him£50.

Past consideration is best described as consideration which hasbeen performed before the parties involved have reached any agreement.The general principle is that past consideration is insufficient tosupport a valid contract. A good illustration is Roscorla v Thomas (1842)where, after the promisee had purchased a horse, the promisor promisedthat the horse was sound. When it transpired that the horse was notsound, it was held that the promisee could not enforce the promise,because all he could show was past consideration. A more modern exampleis Re McArdle (1951) where, after the promisee had spent money onimproving a property, the promisors promised that they would reimburseher. The court held that the promisee was not entitled to the paymentbecause the promise had been made after the work had been done.

There are some exceptions to the above principles. Thus, where thework has been carried out in the course of a business and it was assumedthat the work would be paid for, the consideration will not be past.

An important common law exception arises where a past act has beendone at the request of the promisor and both parties understood that theact was to be paid for. If the promisor subsequently promises a paymentthen it will be enforced by the courts.

Question 6

Raymond has agreed to accept £200 in settlement of a debt of£800. The question is whether Raymond can go back on that agreement andsue for the balance.

The rule in Pinnel's case (1602) states that 'payment of asmaller sum does not discharge a debt of a greater amount'. This isbecause the debtor has not given consideration for the creditor'sagreement to accept the smaller amount – Foakes v Beer (1884).Under the rule Raymond may go back on his agreement to accept £200 infull and final settlement and may sue Samantha for the outstandingbalance of £600.

There are, however, four exceptions to the rule in Pinnel's case (1602).

(1)Accord and satisfaction

Accord means agreement and must be freely entered into by the creditor for his benefit. In D & C Builders v Rees (1966)Mrs Rees owed a sum to the claimants in respect of some building work.Knowing that the claimants were in a parlous financial state, she putpressure on them to accept a smaller sum. Under protest the buildersaccepted the smaller sum. It was held that there was no true accord.Likewise here, Raymond 'after a bitter argument … reluctantly'accepted the £200.

Satisfaction means consideration. Thus if, e.g. the creditoragreed to accept payment of a smaller sum at a date earlier than thewhole debt was due, this benefit would constitute consideration for hispromise to accept the smaller amount. Other examples of considerationinclude the giving of an asset, the performance of a service, anddifferent modes of payment – although in D & C Builders v Rees (1966)it was held that there is no distinction, for these purposes, betweenpayment by cheque and payment by cash. Thus Samantha's payment of £200by cheque confers no benefit when compared to £800 in cash. If theagreement to accept the smaller sum is by deed then it is binding on thecreditor even though the debtor has given no satisfaction. AccordinglyRaymond is advised that there is most probably no 'accord' and certainlyno 'satisfaction' sufficient to prevent him from suing for the £600balance.

(2)Part payment by a third party

If the part payment to the creditor is made not by the debtor butby some third party then the creditor cannot pursue the debtor for thebalance. The given facts state that Samantha paid the £200, not someoneelse; so this exception does not apply.

(3)Composition with creditors

If the agreement with the creditor is part of a wider agreementbetween the debtor and his creditors, and all the creditors accept thedebtor's proposals, they cannot later pursue the debtor for anyshortfall. The given facts do not indicate that Samantha entered intosuch a composition; so this exception does not apply.

(4)Equitable doctrine of promissory estoppel

In certain circumstances, equity will stop a creditor from going back on his promise to accept a smaller amount. In Central London Property Trust v High Trees House (1947)it was established that the doctrine applies only where the creditorhas waived his rights with the intention that the debtor will alter his[legal] position in reliance on the waiver and that as a result of suchreliance it would in all the circumstances be just and equitable to holdthe creditor to his word.

There are limitations to the doctrine:

  • It does not apply to simple debtor-creditor situations as is Samantha and Raymond's situation – Re Selectmove (1995) where the Inland Revenue was not estopped from going back on its agreement to accept overdue tax by instalments. In contrast, in Central London Property Trust v High Trees House (1947) a landlord agreed to reduce the head rent so as to enable the tenant to sub-let at much lower rents during the war.
  • It does not apply where the debtor has acted unfairly such as in D & C Builders v Rees (1966) where the creditor's promise was extracted under pressure. In contrast, in Central London Property Trust v High Trees House (1947) the landlord freely agreed to reduce the head rent. Raymond did not freely agree to waive the full amount.

In summary: Raymond is advised that none of the exceptions to the rule in Pinnel's case (1602) apply and he may therefore sue Samantha for the £600 balance.

Question 7

The issue here is the validity of Office Supplies Ltd's exemptionclause. An exemption clause is a term in a contract which seeks to limitor exclude the liability of one of the parties in the event of abreach. Grace is advised that it will be valid at common law if theclause is incorporated into the contract and is drafted with words thatare apt to cover the loss suffered; and it must not be rendered void byUCTA 1977.

Common law – incorporation into the contract

Clauses can be incorporated into a contract by signature, notice ora consistent course of dealing between the parties. There is noindication on the facts of the problem that there has been a priorcourse of dealing between Grace and Office Supplies Ltd. Neither isthere any evidence that the clause has been incorporated by notice. Theissue therefore is whether the clause has been incorporated bysignature.

If a person signs a document he is bound by its terms even if he has not read it – L'Estrange v Graucob Ltd (1934), unless there is a misrepresentation as to its contents – Curtis v Chemical Cleaning and Dying Co (1951). This is so even if he is incapable of reading it – Thompson v LMS Railway Co (1930). Any signature must be obtained before the making of the contract.

Applying the above to the facts, the clause is incorporated because Grace signed the contract containing the exemption clause.

Common law – the wording of the exemption clause

The second requirement of common law is that in order to be validthe clause must be worded so as to exclude the loss in question. If thewording of the clause is ambiguous, the courts will interpret it contraproferentem, i.e. in the way least favourable to the party seeking torely on it.

Here the words of the clause 'any financial losses or other losses'are wide enough to cover Grace's losses. And although the clause didnot specifically refer to negligence, the words 'however caused' areprobably wide enough to cover negligence.

Statutory test

As this is a contract made in the course of business, UCTA 1977applies. This provides that any exclusion of liability for death orpersonal injury caused by negligence is void. Thus Office Supplies Ltd'sclause does not exempt it from liability for Grace's personal injuriesand therefore she can sue for these losses.

Any exclusion of liability for loss other than death or personalinjury caused by negligence is void unless it satisfies thereasonableness test. Thus Office Supplies Ltd can only rely on itsexemption clause to protect it from liability for Grace's loss ofprofits if it can prove that the exemption is reasonable.

The court adjudges whether the exclusion clause is reasonable inthe light of all the surrounding circumstances and in particular:

  • the relative bargaining strength of the parties. The given facts suggest that Grace ('a sole practitioner') is a 'small man' and may well be in a weak bargaining position. No indication is given as to whether Office Supplies Ltd was in the strong position of being able to dictate its terms to Grace.
  • whether the party seeking to rely on the clause offered an inducement to the other to get him to agree to it. No information is given on the matter.
  • whether the other party knew or ought to have known of the existence and extent of the clause. This factor will operate in Office Supplies Ltd's favour since Grace had the opportunity to read the clear words on the contract she signed.
  • the ability to insure. This factor would appear to be neutral since presumably both Office Supplies Ltd and Grace could insure.

On the bare facts given, it would appear that Office Supplies Ltd'sexclusion clause with regard to Grace's loss of profits is reasonable.Grace is advised that she cannot sue for these losses.

Question 8 - Fixed Test 1

This question is a FIXED TEST. Please answer the question in full, without reference to an answer.

Then log-in to en-gage and answer fixed test 1 based on your full long-form answer

...or alternatively register using the information at the back of your Complete Text and then go to www.en-gage.co.uk

Question 9

The principle remedies for breach of contract are:

Damages

Every failure to perform a primary obligation is a breach ofcontract. The secondary obligation on the part of the contract-breakerto which it gives rise by implication of the common law is to paymonetary compensation to the other party for the loss sustained by himin consequence of the breach (Photo Productions Ltd v Securicor Transport Ltd (1980)).

Monetary compensation for breach of contract is known as damages.The estimation of what damages are to be paid by a party in breach ofcontract can be divided into two parts: remoteness and measure.

I     Remoteness of damage involves deciding how far down a chain of events a defendant is liable (Hadley v Baxendale (1854) and Victoria Laundry Ltd v Newman Industries Ltd (1949)).

II Measure of damages. The object is notto punish the party in breach, so the amount of damages awarded cannever be greater than the actual loss suffered. The aim is to put theinjured party in the same position they would have been in had thecontract been properly performed.

Specific performance

An order for specific performance requires the party in breach to complete their part of the contract.

ISpecific performance will only begranted in cases where the common law remedy of damages is inadequate.It is most commonly granted in cases involving the sale of land, wherethe subject matter of the contract is unique.

II Specific performance will not begranted where the court cannot supervise its enforcement. For thisreason it will not be available in respect of contracts of employment orpersonal service.

IIISpecific performance, as an equitable remedy, will not be granted where the plaintiff has not acted properly on their part.

Injunction

This is also an equitable order of the court, which directs aperson not to break their contract. An injunction will only be grantedto enforce negative covenants within the agreement, and cannot be usedto enforce positive obligations. However, it can have the effect ofindirectly enforcing contracts for personal service (Warner Brothers Pictures Inc v Nelson (1937)).

Rescission

This equitable remedy entitles the innocent party to a voidablecontract to treat it as if it had never been made and consequently torecover all money or assets that had previously been exchanged under thecontract.

Question 10

This question concerns the potential liability for economic losscaused by negligent advice and information. It is more difficult toimpose liability for negligent statements than for negligent actions:careless words can spread rapidly resulting in a proliferation of claimsand potentially unlimited financial losses. Therefore, policyconsiderations have led to the introduction of rules which limit theclass of potential claimants in such cases. The mere foresight of harmarising from negligent advice is insufficient to establish a duty ofcare.

Starting from the House of Lords' decision in Hedley Byrne & Co Ltd v Heller & Partners Ltd (1964) a duty of care arising from negligent misstatement can only be established if the following criteria apply:

AThere must have been a special relationship of close proximity between the claimant and the defendant.

BIt must have been reasonable for the claimant to rely on the defendant's statement.

CIt would be fair, just and reasonablein all circumstances for the courts to impose a duty of care. Thisrequirement is heavily influenced by policy considerations.

Special relationship of close proximity

The existence of such a relationship is dependent upon a variety offactors. It has been suggested that a relationship `equivalent tocontract' must exist between the claimant and the defendant before thedefendant will be held liable to have voluntarily assumed responsibilityto the claimant

In Caparo Industries v Dickman, Lord Oliver defined the range of persons to whom a duty is owed in terms of the purpose for which the statement is made:

(1)The advice must be required for apurpose which is either specified in detail or described in generalterms, and this purpose must be expressly or inferentially made known tothe adviser when the advice is given.

It is arguable here that the information concerning Albert is veryspecific and Charles had voluntarily assumed responsibility to Brad inrespect of the accuracy of this statement.

(2)The advisor must know (expressly orinferentially) that the advice will be communicated to the claimant,either specifically or to a member as an ascertained class in order thatit should be used by the advisee for that purpose. It is submitted thatCharles knew Brad would be relying on him.

(3)It must be expressly or inferentiallyknown that the advice communicated is likely to be acted upon by theadvisee for that purpose without independent inquiry.

(4)The advice must be acted upon by the advisee to his detriment. Brad did rely on Charles' statement to his detriment.

(5)It must have been reasonable for the claimant to rely on the defendant's statement.

In Hedley Byrne, the appellants were advertising agents, whohad contracted to place advertisements for their client's (Easipower)products. As this involved giving Easipower credit, they asked therespondents, who were Easipower's bankers, for a reference as to thecreditworthiness of Easipower. Heller gave favourable references (butstipulated that the information was given without responsibility ontheir part). Relying on this information, the claimants extended creditto Easipower and lost over £17,000 when the latter, soon after, wentinto liquidation. The claimants sued Easipower's bankers for negligence.

In this scenario Charles had supplied the reference with adisclaimer. As such this disclaimer was adequate to exclude theassumption by him of the legal duty of care.

It is unlikely that Brad would succeed in an action againstCharles. However, in the absence of the disclaimer, the circumstanceswould have given rise to a duty of care in spite of the absence of acontractual or fiduciary relationship.

Question 11

This question requires discussion of the legal requirements fornegligence and proximity, then a discussion of whether there is a'special relationship' involving an 'assumption of responsibility' asrequired in Hedley Byrne & Company v Heller & Partners.

Mary v Barnett Hospital

Can it be said that Barnett Hospital are in the business ofproviding references and are they aware that Comfort Home will act onit? Strictly speaking no, but it could be argued that it is part of anemployer's function to provide references.

Can Caparo be applied?

  • The advice is required for a purpose made known to the adviser, either actually or inferred by the circumstances;
  • The adviser knows that his advice will be communicated to the recipient, either specifically or as a member of an ascertainable class; He knows that the advice is likely to be acted upon by the claimant without independent enquiry; and
  • The advice is acted upon to the claimant's detriment.

Spring v Guardian Assurance – the decision here isdirectly applicable to the problem. The House of Lords felt that therewas sufficient proximity between an ex-employer and ex-employee and thatit was just, fair and reasonable to impose a duty.

Mary v Joe

Although Joe is an investment advisor, is he aware that Maryintends to act on his advice? Although as the advice has been given in asocial setting, rather than a business setting, it could be argued thathe assumed responsibility by claiming to know about investments.

It will also be necessary to test whether there has been reliance.

Were the shares bought solely on Joe's advice or were theresentimental reasons for choosing to purchase shares in her old school?

If Mary can get round the problems of the social setting andreliance, the fact that the advice was given face to face may support anargument that there is an assumption of responsibility on Joe's part.

Mary v Thomas

Again this would involve a discussion of whether there is 'specialskill' and 'reasonable reliance'. It is unlikely that there is 'specialskill' as Thomas is not in the business of giving investment advice,neither is he aware that Mary was seeking considered advice and intendedto act on it. Further no duty would arise following the decision in Caparo Industries v Dickman.This now contains the general position on when a duty of care arisesfor misstatements. Lord Oliver stated that a duty will arise where:

  • The advice is required for a purpose made known to the adviser, either actually or inferred by the circumstances;
  • The adviser knows that his advice will be communicated to the recipient, either specifically or as a member of an ascertainable class; He knows that the advice is likely to be acted upon by the claimant without independent enquiry; and
  • The advice is acted upon to the claimant's detriment.

Thomas knows nothing about Mary or the fact that Mary read hisarticle with the intention of acting upon it. Given the number ofindividuals likely to read the article, it would also not be 'just, fairand reasonable' to impose a duty of care on Thomas in thecircumstances.

Question 12

(Part A)

Employees are people working under a contract of service. Those whowork under a contract for services are independent contractors. Theyare not employees, but are self-employed.

The courts have developed tests for distinguishing the employee from the self-employed.

The first test to be applied by the courts was known as the controltest. The key element in this test is the degree of control exercisedby one party over the other. The question to be determined is the degreeto which the person who is using the other's services actuallycontrols, not only what they do, but how they do it. An example of theuse of this test can be seen in Walker v Crystal Palace Football Club (1910)in which it was held that a professional football player was anemployee of his club, on the ground that he was subject to control inrelation to his training discipline and method of payment.

The control test looks back to and reflects the previousmaster/servant relationship of employment, but its main shortcoming liesin its lack of any degree of subtlety. Highly-skilled professionals,such as surgeons, by necessity have a high level of control over howthey perform their day-to-day work, and under the control test, theywere deemed to be self-employed rather than employees. Consequently,patients who had suffered as a consequence of negligence would only beable to sue the doctor, rather than the health authority which used thesurgeons' services. Such weakness in the control test led to the courtsdeveloping a more subtle test.

The integration test shifted the emphasis from the degree ofcontrol exercised over an individual to the extent to which theindividual was integrated into the business of his putative employer. Anexample of the application of the integration test may be seen in Whittaker v Minister of Pensions & National Insurance (1966)in which the court found that the degree to which a circus trapezeartist was required to do other general tasks in relation to theoperation of the circus in which she appeared, indicated that she was anemployee rather than selfemployed. As a consequence, she was entitledto claim compensation for injuries sustained in the course of heremployment. However, even the integration test was not without problems,as some employers attempted to give the impression of using aself-employed workforce whilst effectively still controlling what thatworkforce did.

The response on the part of the courts was the development of themultiple, or economic reality, test. Rather than relying on one singlefactor, this test uses a more general assessment of the circumstances ofany particular case in order to decide whether someone is an employee.In so deciding, the courts will not be bound by how the partiesthemselves describe the relationship. Thus it is immaterial that theagreement between the parties states that someone is to beself-employed. If the indications are otherwise then the person will berecognised, and treated, as an employee (Market Investigations Ltd v Minister of Social Security (1968)).

The economic reality test was first established in Ready Mixed Concrete (South East) Ltd v Minister of Pensions & National Insurance & Others (1968) in which it was held that there were three conditions supporting the existence of a contract of employment:

Ithe employee agrees to provide his own work and skill in return for a wage

II the employee agrees, either expressly or implicitly, that he will be subject to a degree of control, exercisable by the employer

IIIthe other provisions of the contract are consistent with its being a contract of employment.

In deciding whether or not there is a contract of employment, thecourts tend to focus on such issues as whether wages are paid regularlyor by way of a single lump sum; whether the person receives holiday pay;and on who pays national insurance and income tax. However, there canbe no definitive list of tests, as the whole point of the multiple testis that it examines all aspects of the situation in order to reach adetermination. For example in Nethermere (St Neots) Ltd v Taverna and Gardiner (1984),a group of home workers (people who carried out paid work in their ownhomes) were held to be employees on the grounds that they were subjectto an irreducible minimum obligation to work for their employer.

(Part B)

Applying the economic reality test, it is more than likely that Danwould be treated as an employee, but Eve would be treated asself-employed. It is true that they were both described asself-employed, but it should be recognised that the label applied doesnot by itself define the relationship (Market Investigations Ltd v Minister of Social Security (1968)).Looking at the circumstances, it can be seen that the manner in whichthey paid tax might indicate that they were self-employed, but the factthat Carol provided them with their equipment suggests that they wereemployees. In the final analysis, the most significant factor wouldappear to be the degree to which Carol controlled them. Dan had to workfor Carol only and on her premises, whereas Eve not only was allowed towork for others but, most importantly, she was also allowed to useothers to do her work for Carol. This suggests clearly that Eve was notemployed by Carol, although Dan was.

Question 13

AThe essential issue here is whetherNorman is able to terminate his contract with Allpass Ltd, butnevertheless argue for the purposes of the legislation providingprotection against unfair dismissal that he has been, in law,'dismissed'.

The definition of dismissal is contained in s95 ERA 1996 which provides that an employee is treated as dismissed if:

Ithe contract under which he is employed is terminated by the employer (whether with or without notice), or

II he is employed under a contract for a fixed term and that term expires without being renewed under the same contract, or

IIIthe employee terminates thecontract under which he is employed (with or without notice) incircumstances in which he is entitled to terminate it without notice byreason of the employer's conduct.

Since the decision in Western Excavating (ECC) Ltd v Sharp (1978)it has been clear that an employee can argue that he has beenconstructively dismissed, by virtue of s95(1)(c), in situations wherethe employer has breached the contract in such a way as to justify theemployee in treating himself as discharged from future performance. Onthe facts of the problem, Norman will be able to regard himself asconstructively dismissed if Allpass Ltd's actions in requiring him towork in Bristol rather than Manchester are a sufficiently serious breachof Norman's contract such as to entitle Norman to treat himself asdischarged from future performance.

BIf the employment tribunal is satisfiedthat Norman was constructively dismissed from his employment, thenNorman will be able to pursue an unfair dismissal claim. He has morethan the necessary qualifying period of service – one year – but, inorder for him to succeed, the employment tribunal will need to besatisfied that the employer's reasons underlying the dismissal were notsuch as to justify it, or that the employer had acted unreasonably intreating the reason/s as sufficient.

There are several categories of reasons which may be used by anemployer to justify dismissal, but the one which is relevant in thepresent instance relates to redundancy: if an employer dismisses anemployee for reasons of redundancy, then that may be a fair reason fordismissal (as long as the employer, in addition, acts reasonably).'Redundancy' covers situations where the employer's requirement for workof a particular kind to be done in the place where the employee isemployed has ceased or diminished. The facts of the problem make itclear that there has been such a diminution in the employer'srequirements. The indications are that Norman's place of work isManchester (as opposed, for example, to the whole of the UK). Primafacie, therefore, a redundancy situation has arisen. The essentialquestion will be whether Allpass Ltd has acted reasonably.

Tribunals have not come too readily to the conclusion thatdismissals based on redundancy are unfair, but this does not abrogatethe responsibility on employers to act reasonably in dismissing forredundancy. The kinds of factors which are relevant are the period ofwarning of impending redundancy which has been given, whether objectivecriteria have been applied in selecting a person for redundancy andwhether an employer has applied these fairly, whether there has beenconsultation with the employee and whether the employer has investigatedwhether it is possible to offer an employee alternative employmentrather than dismiss him. Norman would need to be advised that hischances of success in an unfair dismissal action will depend on whetherAllpass Ltd has considered such factors before deciding on Norman'srelocation.

Question 14

Agency of necessity

A person who acts to save the property of another, or gives himsome other form of assistance, may as a matter of law be regarded as anagent of necessity. For example, a person who goes to the aid of a shipat sea and saves life or property is entitled to a reward. The amount ofthe reward is at the discretion of the court. The doctrine of agency ofnecessity is confined to fairly narrow limits. In general threeconditions must be satisfied.

I    There must be an emergency, making it necessary for the agent to act as he did. In Pager v Blatspiel,A bought skins as agent for P but was unable to send them to P becauseof prevailing war conditions. Since A was also unable to communicatewith P, he sold the skins before the end of the war. It was held that Awas not an agent of necessity, because he could have stored the skinsuntil the end of the war. There was no real emergency.

II    It must be impossible to get instructions from the principal. In Springer v Great Western Railway,a consignment of fruit was found by the carrier to be going bad. Thecarrier sold the consignment locally instead of delivering it to itsdestination. It was held that the carrier was not an agent of necessitybecause he could have obtained new instructions from the owner of thefruit. He was therefore liable in damages to the owner.

III   The agent must have acted in good faith, and in the interests of the principal. In Great Northern Railway v Swaffield (1874)a horse was sent by rail and on its arrival at its destination therewas no one to collect it. GNR incurred the expense of stabling the horsefor the night. It was held that GNR was an agent of necessity, whotherefore had authority to incur the expense in question.

Agency of ratification

If a duly appointed agent exceeds his authority, or a person havingno authority purports to act as agent, the principal is not bound. Theprincipal may however adopt the contract at a later date, provided:

I    The agent named or otherwise identified his principal and the third party knew that the agent was contracting as agent. In Keighly Maxsted v Durant,A was authorised by P to buy wheat at a certain price. In excess of hisauthority, A bought from T at above that price. A professedlycontracted in his own name, without disclosing the agency to T. The nextday P purported to ratify. It was held that P could not ratify because Ahad neither named nor otherwise identified P and T did not know he wasdealing with an agent.

II   The principal had contractual capacity at the date of both thecontract and the ratification. If the principal is a company, thismeans it must have been incorporated at the time of the contract. In Kelner v Baxter (1866)P sold wine to D who purported to act as agent for a company which wasabout to be formed. When it was formed the company attempted to ratifythe contract made by D. It was held that it could not do so, since itwas not in existence when the contract was made.

III  The principal had full knowledge of all material facts, or was prepared to ratify in any event. For example, in Fitzmaurice v Bayley,a principal in effect said to a third party – I don't know what myagent has agreed to do, but I must support him. It was held that he hadratified. Although he did not know that the agent had exceeded hisauthority, he had agreed to take the risk.

IV  The principal must ratify the whole contract, not merely parts of it.

V   The principal must ratify within a reasonable time.

VI  The principal must communicate a sufficiently clear intention of ratifying, either by express words or by conduct.

A void or illegal contract cannot be ratified because it is of no legal effect from its inception.

It can therefore be seen that the main difference between creationof agency by ratification and creation by necessity is that agency ofnecessity is created by operation of law without the agreement of theprincipal, whereas agency by ratification can only be created with theknowledge and consent of the principal.

Question 15

Dan

The rules relating to the residual responsibility of retiredpartners for partnership debts depend on when the debts were contractedand the action taken by the former partner to announce their retirementfrom the business.

A retired partner remains liable for any debts or obligationsincurred by the partnership prior to retirement. Thus the date of anycontract determines responsibility: if the person was a partner when thecontract was entered into, then they are responsible, even if thecontract is completed after their retirement. It is possible for theretiring partner to be discharged from existing liability though as aconsequence of a contract of novation.

Novation is essentially a tripartite contract involving theretiring partner, the remaining members of the continuing partnershipand the existing creditors. Under such an agreement any liability of theretiring partner is passed to the remaining partners. As creditorseffectively give up rights against the retiring partner, their approvalis required. Such approval may be express or it may be implied from thecourse of dealing between the creditor and the firm.

Where someone deals with a partnership after a change inmembership, they are entitled to treat all the apparent members of theold firm as still being members until they receive notice of any changein the membership.

In order to avoid liability for future contracts, a retiringpartner must ensure that individual notice is given to existingcustomers of the partnership; and advertise the retirement in the LondonGazette. This serves as general notice to people who were not customersof the firm prior to the partner's retirement, but knew that thatperson had been a partner in the business. Such an advert is effectivewhether or not it comes to the attention of third parties.

It follows from this that Dan could be liable for any debts towardsthe longstanding customer, Greg, unless he has taken steps to notifyGreg of his retirement from the partnership, which does not appearlikely.

Frank

Frank had let it be known generally that he was a partner and if,as would appear likely, the other partners knew about Frank's claim anddid nothing to deny it, then they would be estopped subsequently frominsisting on the true nature of affairs (Freeman and Lockyer v Buckhurst Park Properties Ltd (1964)).Frank would therefore be seen as a partner with the authority to bindthe partnership (s.5 Partnership Act 1890). However, the partnershipwould be liable for the contracts even if the other partners were notaware of Frank's claim to be a partner. The question states that Eve andClare left much of the day to day running of the business to Frank andit can be seen that, on that basis alone, he had the authority to managethe business irrespective of the question as to whether he was a memberof the partnership or not. Third parties are entitled to assume thatagents holding a particular position have all the powers that areusually provided to such an agent. This is referred to as implied actualauthority and means that, without actual knowledge to the contrary,outsiders may safely assume that an agent has the usual authority thatgoes with their position (Watteau v Fenwick (1893)). Enteringinto ordinary trading contracts, such as the one with Greg, would comewithin Frank's implied actual authority as the business manager.

As for Frank's liability, anyone who represents themselves, orknowingly permits themselves to be represented, as a partner is liableto any person who gives the partnership credit on the basis of thatrepresentation. The partners would be estopped from denying Frank'smembership if they knew of his claim to be a partner. Frank would alsobe estopped from denying that he was a partner. Frank therefore wouldalso be liable for the debts.

Clare and Eve being active partners have full responsibility for the partnership debts.

Under s9 of the PA 1890, the liability of partners as regards debtsor contracts is joint and several. The effect of joint liability usedto be that, although the partners were collectively responsible, aperson who took action against one of the partners could take no furtheraction against the other partners, even if they had not recovered allthat was owing to them. That situation was remedied by the CivilLiability (Contributions Act) 1978. This act effectively states that ajudgment against one partner does not bar a subsequent action againstthe other partners. This means that as regards Greg's debt Clare, Dan,Eve and Frank are all personally responsible for any shortfall and hemay take action against any one of them. The one against whom the actionis taken will be able to claim a proportionate indemnity from theothers.

Question 16

Grounds for dissolution

Partnerships are created by agreement and may be brought to an endin the same way. However, subject to any provision to the contrary inthe partnership agreement, the PA 1890 provides for the dissolution of apartnership on the following grounds:

The expiry of a fixed term or the completion of a specified enterprise

It is possible for a partnership to be established for a statedperiod of time and at the end of that time the partnership will come toan end and the partnership will be dissolved. Alternatively it ispossible for the partnership to be established in order to achieve aparticular goal and again once that goal has been attained thepartnership will come to an end.

The giving of notice

If the partnership is of indefinite duration, then it can bebrought to an end by any one of the partners giving notice of anintention to dissolve the partnership.

The death or bankruptcy of any partner

As in English law the ordinary partnership has no legal personalityin its own right, but merely exists as a collection of individuals, itis apparent that the death of a member will bring about the end of thepartnership (n.b. this of course is not the case with limitedpartnerships formed under the Limited Liability Partnerships Act 2000which does provide legal capacity to such partnerships formed under itsprovisions). The bankruptcy of a partner has the same effect.

Although the occurrence of either of these events will bring theoriginal partnership to an end, it is usual for partnership agreementsto provide for the continuation of the business under the control of theremaining/solvent partners who will constitute a new partnership.

Illegality

The occurrence of events making the continuation of the partnershipillegal will bring it to an end. This is illustrated by the case of Hudgell, Yeates and Co v Watson (1978).Practising solicitors are legally required to have a practicecertificate. However, one of the members of a three-person partnershipforgot to renew his practice certificate and thus was not legallyentitled to act as a solicitor. It was held that the failure to renewthe practice certificate brought the partnership to an end, although anew partnership continued between the other two members of the oldpartnership.

By court order

In addition to the provisions listed above, the court may, mainlyby virtue of s35 of the PA 1890, order the dissolution of thepartnership under the following circumstances:

IWhere a partner becomes a patient under the Mental Health Act 1893.

II Where a partner suffers some other permanent incapacity.

This provision is analogous to the previous one. It should benoted that it is for the other partners to apply for dissolution andthat the incapacity alleged as the basis of dissolution must bepermanent.

IIIWhere a partner engages in activity prejudicial to the business.

Such activity may be directly related to the business, such asthe misappropriation of funds. Alternatively, it may take place outsidethe business but operate to its detriment. An example might be acriminal conviction for fraud.

IVWhere a partner persistently breaches the partnership agreement.

This provision also relates to conduct which makes itunreasonable for the other partners to carry on in business with theparty at fault.

IVWhere a partner persistently breaches the partnership agreement.

This provision also relates to conduct which makes itunreasonable for the other partners to carry on in business with theparty at fault.

IVWhere the business can only be carried on at a loss.

This provision is a corollary of the very first section of the PA1890 in which the pursuit of profit is part of the definition of thepartnership form. If such profit cannot be achieved, then the partnersare entitled to avoid loss by bringing the partnership to an end.

VIWhere it is just and equitable to do so.

The courts have wide discretion in relation to the implementationof this power. A similar provision operates within company legislation(s.122 Insolvency Act 1986) and the two provisions come together in thecases involving quasipartnerships (Re Yenidje Tobacco Co Ltd (1916) and Ebrahimi v Westbourne Galleries Ltd (1973)).

Question 17

Except in relation to specifically exempted companies, such asthose involved in charitable work, companies are required to indicatethat they are operating on the basis of limited liability. Thus privatecompanies are required to end their names, either with the word'limited' or the abbreviation 'ltd', and public companies must end theirnames with the words 'public limited company' or the abbreviation'plc'. Welsh companies may use the Welsh language equivalents (CompaniesAct (CA) 2006 ss58, 59 & 60).

Although there is no longer an official Business Names Registry,the Registrar of companies maintains a register of business names, andwill refuse to register any company with a name that is the same as onealready on that index (CA06 s66(1)). This control is less rigorous thanthat exercised under the previous legislation and has led to an increasein the use of the tort of 'passing off', as a means of protecting thegoodwill attached to particular business names (see (b) below).

Certain categories of names are, subject to the decision of the Secretary of State, unacceptable per se, as follows:

  • Names which in the opinion of the Secretary of State constitute an offence (s53(a)). As an example, it is illegal for non-designated businesses to claim to be banks, but the powers of the Secretary of State are wide enough to control names which might be considered as inciting race hatred.
  • Names which in the opinion of the Secretary of State are offensive (s53(b)).
  • Names which are likely to give the impression that the company is connected with either government or local government authorities (s54(1)).
  • Names which include a word or expression specified under the Company and Business Names Regulations 1981 (s55(1)).

This category requires the express approval of the Secretary ofState for the use of any of the names or expressions contained on thelist, and relates to areas which raise a matter of public concern inrelation to their use.

Under s67 of the Companies Act 2006 the Secretary of State haspower to require a company to alter its name under the followingcircumstances:

  • Where it is the same as a name already on the Registrar's index of company names.
  • Where it is 'too like' a name that is on that index.

The name of a company can always be changed by a special resolutionof the company so long as it continues to comply with the aboverequirements (s77).

Although a company's name must not be the same as any alreadyregistered, the Business Names Act 1985 does not prevent one businessfrom using the same, or a very similar, name as another business.However, the tort of passing off prevents one person from using any namewhich is likely to divert business their way by suggesting that thebusiness is actually that of some other person or is connected in anyway with that other business. It thus enables people to protect thegoodwill they have built up in relation to their business activity. In Ewing v Buttercup Margarine Co Ltd (1917) the plaintiff successfully prevented the defendants from using a name that suggested a link with his existing dairy company.

Question 18

AModel articles of association areprescribed by the Secretary of State, although companies may alter themodels to suit their particular circumstances and requirements. It isusual for companies to draw up their own particular articles, but ifthey elect not to draw up their own, the model articles applyautomatically. The model articles also apply to the extent that thecompany's articles have not expressly excluded their provisions. Themodel articles cover such matters as the issue and transfer of shares,the rights attaching to particular shares, the rules relating to theholding of meetings, the powers of directors, and the payment ofdividends.

BSection 33 of the Companies Actprovides that the provisions of a company's constitution (i.e. itsarticles) 'bind the company and its members to the same extent as ifthere were covenants on the part of the company and each member toobserve those provisions'. This section has three effects.

I    The documents establish a contract which binds each member to the company. Thus in Hickman v Kent or Romney Marsh Sheep-Breeders' Association (1915),the company was able to enforce an article against a member thatprovided that disputes involving the member and the company should go toarbitration.

II    The company is contractually bound to each of its members. On this basis in Pender v Lushington (1877) a member was able to sue in respect of the wrongful denial of his right to vote at a company meeting.

III   The articles constitute a contract between the members. In Rayfield v Hands (1960),the articles of the company provided that, where shareholders wished totransfer their shares, they should inform the directors of the company,who were obliged to take the shares equally between them at fair value.When the directors refused to purchase the plaintiff's shares, thecourt held that the directors were bound as members by the articles andtherefore had to comply with the procedure set out there.

Articles only operate as a contract in respect of membership rightsand obligations. Consequently it has been held that, although memberscan enforce them, non-members, or members suing in some other capacitythan that of a member, will not be able to enforce promises establishedin the company's articles. In Eley v Positive Government Security Life Assurance Co (1876),the articles of a company stated that the plaintiff was to be appointedas the company's solicitor. It was held that Eley could not use thearticles to establish a contract between himself and the company asthose articles only created a contract between the company and itsmembers. Although Eley was in fact a member, he was not suing in thatcapacity but in the capacity of solicitor, which was not a membershipright.

CSection 21 of the Companies Act 2006allows companies to alter their articles by passing a specialresolution. It is, however, possible to entrench certain provisions.This means that they can only be altered by following a specifiedprocedure, such as obtaining unanimous consent: s22.

Any attempt in the articles to provide that a particularprovision is unalterable is ineffective. The articles cannot be altered,however, in such a way as to conflict with any provision of theCompanies Act. In particular, no member can be bound by any alterationto subscribe for more shares or increase their liability in any way(s25).

Any alteration must be made 'bona fide in the interest of thecompany as a whole', although the exact meaning of this phrase is notaltogether clear. It is evident that it involves a subjective element inthat those deciding the alteration must actually believe they areacting in the interest of the company. There is additionally, however,an objective element. In Greenhalgh v Arderne Cinemas Ltd (1951) it was stated that any alteration had to be in the interest of the 'individual hypothetical member'.

In Brown v British Abrasive Wheel Co (1919) an alterationto the articles of the company was proposed to give the majorityshareholders the right to buy the shares of the minority. It was heldthat the alteration was invalid as it would benefit the majorityshareholders rather than the company as a whole. However, in Sidebottom v Kershaw Leese & Co (1920),an alteration to the articles gave the directors the power to requireany shareholder, who entered into competition with the company, totransfer their shares to nominees of the directors at a fair price. Itwas held that under those circumstances the alteration was valid as itwould benefit the company as a whole.

Question 19 - Fixed Test 2

This question is a FIXED TEST. Please answer the question in full, without reference to an answer.

Then log-in to en-gage and answer fixed test 1 based on your full long-form answer

...or alternatively register using the information at the back of your Complete Text and then go to www.en-gage.co.uk

Question 20

A  The judge in Re Yorkshire Woolcombers' Association (1903) stated that a floating charge has the following characteristics:

  • it attaches to a class of asset, both present and future
  • the assets within the class will be changing from time to time
  • until some step is taken which crystallises the charge, the company remains free to deal with the assets in the ordinary course of business.

BFloating charges have the following disadvantages which may mean that they provide inadequate security:

  • Priorities: fixed charges take priority over floating charges even though they may have been created later.
  • The company's freedom to deal with assets subject to a floating charge includes the ability of the company to dispose of the assets. This may mean that if a liquidation occurs there are few assets available to the floating chargee.
  • On liquidation the claim of the floating chargee is subject to the prior claims of any fixed chargees on the same assets and the preferential creditors.

CWealthy should check the register ofcharges at Hardup plc's registered office and Companies House. In orderto be valid, all charges must be registered at Companies House within 21days of their creation. It is Hardup plc's duty to register the charge,but anyone interested (i.e. Wealthy plc) may affect the registration.If unregistered, the charge is void as against the liquidator, i.e.Wealthy plc becomes an unsecured creditor if Hardup plc goes intoinsolvent liquidation.

Question 21

S168 CA 2006 states that a director can be removed from office atany time by ordinary resolution (i.e. only a simple majority isrequired). The resolution requires special notice (28 days) and Juliecan require the company to circulate written representations to members.S168 therefore overrides the provision in ABC Ltd's articles andJulie's service contract. Provided the procedure was followed correctly,the members had the necessary authority to remove Julie.

As regards the provision in Julie's service contract, if thecontract has been properly approved by the members, she will have anaction against the company for breach of contract. However, if it hasnot been approved and the correct procedures under s168 have beencarried out, the contract has been properly brought to an end and noaction will lie for breach.

As regards the provision in the articles, s33 CA 2006 states thatthe articles are a contract between the company and its members in theircapacity as members. The situation is similar to Eley v Positive Government Security Life Assurance Co (1876)in which it was held that a provision in the articles stating that Eleywas to be the company's solicitor was not enforceable as it was not aright given to him as a member. Therefore, as Julie is affected by thebreach of the articles in her capacity as a director, she cannot sue thecompany for breach of the articles.

Question 22

AThe purchase of the machinery from Nim Ltd

Through his majority interest in Nim Ltd, Len has an interest in the contract with Mod plc.

This means that he should have declared his interest to the boardof Mod plc at the meeting at which the contract was discussed in orderto comply with s175 CA 2006. He should not have voted at the boardmeeting at which the contract was approved, nor should he have beencounted in any necessary quorum for the meeting. There is therefore aclear breach of duty by Len.

Accordingly, Owen is advised that Mod plc (rather than Owenpersonally) has grounds for an action against Len to make him accountfor any profit he made on the deal. It may also be possible to avoid thecontract with Nim Ltd.

A company is a connected person of a director if the directorcontrols more than 20% of its shares. Nim Ltd is therefore a connectedperson of Len. If the value of the machinery was more than £100,000,the contract was substantial and this means that it should have been putbefore the members of Mod plc so as to comply with s190 CA 2006.

If s190 is applicable, breach of it renders the contractvoidable. It also renders Len and Nim Ltd liable to account for anyprofit made or to indemnify Mod plc for any loss. Again, any such actionmust be brought by Mod plc, not Owen personally.

BThe development of the new product by Nim Ltd

Under s177 CA06, Len is under a statutory duty to declare hisinterest in any proposed transaction or arrangement. He should thereforehave declared the extent and nature of his interest to the otherdirectors. This declaration can be made in writing, at a board meetingor by a general notice that he has an interest in a third party.

The situation is similar to that in the case of IDC v Cooley (1972),in which a director was made to account for the profit he made on acontract taken personally because the opportunity to take that contractarose through his directorship. It made no difference that the companywould not have been offered the contract by the other party.

However, the facts here are not the same as IDC in thathere the board of Mod plc has decided not to develop the new product andtherefore Mod plc is not actively pursuing the opportunity.

In Peso Silver Mines v Cropper (1965) it was held that adirector was not in breach of duty where he took an opportunitypersonally after it had been rejected in good faith by his board. Theboard of Mod plc has rejected the opportunity. The difficulty is whetheror not this decision was reached in good faith.

This will not be the case if Len attempted to persuade the boardto reject the opportunity because he wished to take it personally, or,indeed, if he did not use his knowledge and experience for the benefitof Mod plc and thus failed to point out to the board why he liked thenew product.

If Len is accountable, any action to recover the profit must be brought by Mod plc, not Owen personally.

Question 23 - Fixed Test 3

This question is a FIXED TEST. Please answer the question in full, without reference to an answer.

Then log-in to en-gage and answer fixed test 1 based on your full long-form answer

...or alternatively register using the information at the back of your Complete Text and then go to www.en-gage.co.uk

Question 24

ARequirements for companies in respect of holding AGMs

S336 Companies Act 2006 requires every public company to hold an AGM within the six months following their financial year end.

If an AGM is not held in accordance with s336 CA06, the company and every officer in default are liable to a fine.

Private companies are not required to hold an AGM.

BWritten resolution

A private company can use a written resolution to pass any typeof resolution (ie, ordinary or special) which would otherwise need to bepassed at a general or class meeting.

This is subject to two exceptions: removal of directors andremoval of auditors – both require a meeting to be convened so thatthe person concerned can exercise his rights to attend and speak on theresolution.

The resolution is achieved once a copy has been signed by therequisite percentage of members who would be entitled to attend and voteat a meeting. The resolution takes effect from the date the last personsigns.

A copy of the proposed written resolution must be forwarded tothe company's auditor (if any). Failure to do so is a criminal offence,but failure does not affect the validity of the resolution.

All written resolutions must be recorded in the company's Register of written resolutions.

Question 25

ASection 84 IA states that a company may be wound up voluntarily:

Iwhen any period fixed for theduration of the company by the articles expires or any event occurswhich shall, according to the articles, lead to its dissolution. Undersuch circumstances the winding up has to be approved by an ordinaryresolution.

II for any other reason whatsoever. Under these circumstances a special resolution is required to approve the winding up.

In either case the winding up is deemed to have started on the date that the appropriate resolution was passed.

There are two distinct forms of voluntary liquidation:

IMembers' voluntary winding up

This takes place when the directors of the company are of theopinion that the company is solvent and is capable of paying off itscreditors. The directors are required to make a formal declaration tothe effect that they have investigated the affairs of the company andthat in their opinion it will be able to pay its debts within 12 monthsof the start of liquidation. It is a criminal offence for directors tomake a false declaration without reasonable grounds. On appointment, byan ordinary resolution of the company, the job of the liquidator is towind up the affairs of the company, to realise the assets and distributethe proceeds to its creditors. On completion of this task theliquidator must present a report of the process to a final meeting ofthe shareholders. The liquidator then informs the Registrar of theholding of the final meeting and submits a copy of his report to it. TheRegistrar formally registers these reports and the company is deemed tobe dissolved three months after that registration.

II Creditors' voluntary winding up

This takes place when the company is insolvent when it is decidedto wind it up. The essential difference between this and the formertype of winding up is that, as the name implies, the creditors have anactive role to play in overseeing the liquidation of the company.Firstly a meeting of the creditors must be called within 14 days of theresolution to liquidate the company at which the directors must submit astatement of the company's affairs. The creditors have the final say inwho should be appointed as liquidator and may, if they elect, appoint aliquidation committee to work with the liquidator. On completion of thewinding up the liquidator calls and submits his report to meetings ofthe members and creditors.

The liquidator then informs the Companies' Registry of theholding of these final meetings and submits a copy of his report to it.The Registrar formally registers these reports and the company is deemedto be dissolved three months after that registration.

Question 26

AThe Cadbury Committee examinedboardroom accountability issues and one of its solutions was themonitoring role of non-executives directors.

Cadbury recommended that there should be sufficient independentNEDs in number and quality to carry significant weight in boarddecisions. These NEDs should be independent of the company.

The NEDs should sit in sub-committees of the board to coverissues concerning appointments of directors, remuneration of directorsand the audit process. The role of the NEDs is to bring judgement andexperience to the board that the executive directors might lack. Incontrast to executive directors, NEDs do not usually have a fulltime-relationship with the company. They are not employees and onlyreceive directors' fees. They are expected to exert a measure of controlover the executive directors to ensure that they run the company in thecompany's best interest. NEDs should scrutinise the performance ofmanagement in meeting goals and objectives, and monitor the reporting ofperformance. As far as company law is concerned there is no distinctionbetween the executive directors and NEDs; both are subject to the samecontrol and liabilities.

The following are evidence in favour of the presence of NEDs on boards.

There is evidence that non-executive directors perform animportant corporate governance function. Without the monitoring functionof NEDs, it would be more likely that executive directors would be ableto manipulate their position by gaining complete control over their ownremuneration packages and securing their jobs.

Without NEDs, executive directors can become excessively cohesive.

The addition of the NEDs brings new information and ideas and allows the entire board to make sound decisions.

NEDs provide an independent and fresh review of long termdecisions, effectuate impartial, uncontaminated audits of managerialperformance and counterbalance the influence of top management.

NED's independent influence has led to the removal of ineffective chief executives.

However, there is also a perception that the involvement ofnon-executive directors can damage corporate governance by reducingentrepreneurship in the business and weakening board unity.

This was certainly the view expressed by many board directors intheir initial response to the Higgs recommendations to broaden the roleand effectiveness of non-executive directors in the UK. Higgsrecommended that a senior independent director should be identified andthis senior NED should be available to shareholders if their concernshave not been met through normal channels of contact.

There was also potential for the appointment of non-executivedirectors to result in more cronyism and a more comfortable network ofclose ties and cosy relationships between the directors of leadingcompanies.

Accusations were made that the new level of non-executivedirectors provides just more 'jobs for the boys' and the opportunity foreven firmer golden handshakes than retiring directors receive already.Nonetheless despite these criticisms, the presence of independentnon-executive directors is necessary to improve the quality ofgovernance in listed companies.

BThe Higgs review has provided detailed guidance on the role of the NEDs.

It specified the number of independent NEDs on a main board and crucially gives a good definition of independence.

Below are the key recommendations of Higgs to ensure the independence of NEDs.

(1)At least half the board, excluding the chairman, should be independent NEDs.

(2)Higgs sets out a definition ofindependence. A NED is considered independent when the board determinesthat the director is independent in character and judgement and thereare no relationships or circumstances which could affect, or appear toaffect, the director's judgment. Such relationships and circumstancesarise where the director is or has been an employee of the company, hasor had a business relationship with the company, is being paid by thecompany other than a director's fee and certain other payments; hasfamily ties to the company or its employees; holds cross-directorshipsor has significant links with other directors through involvement inother companies or bodies; represents a significant shareholder, hasserved on the board for 10 years.

(3)The board should identify in itsannual report the NEDs it determines to be independent. The board shouldstate its reasons if a director is considered independentnotwithstanding the existence of relationships or circumstances whichmay appear relevant to its determination.

Higgs contribution is significant. Its key weakness is perhapscompanies get to say who is independent or not according to the Higgscriteria without additional external scrutiny.

CThe Combined Code is a set ofprinciples, rather than a set of rules. It requires the directors todescribe in their own words the way in which they have applied thegeneral principles of corporate governance.

Self-regulation: advantages

  • Legislation may be too rigid and difficult to change to keep pace with corporate change. A 'one size fits all' piece of legislation may be based on the 'lowest common denominator' and hinder rather than improve good governance because of the diversity amongst corporations.
  • The present system is flexible and provides for responsiveness to change. This means that the Combined Code can be updated to respond to changing conditions and changing expectations of shareholders and others.
  • Because the directors report on the actual circumstances of their own company, the report should be more meaningful than one based on specific detailed requirements.
  • A principles-based approach encourages the directors to follow the spirit of the Code; whereas a rules-based approach may result in tick-box mentality. This means that, under a rules-based approach, the directors may follow the letter of the rules rather than their spirit.

Self-regulation: disadvantages

  • Bias – favouring the interest of members at the expense of other interests.
  • Prevents coherent reforms by government as self-regulatory practices will have developed and been relied upon.
  • Enforcement problems – insufficient investigatory and enforcement powers and difficulties of co-ordination.

Question 27

Under the Criminal Justice Act (CJA) 1993, insider dealing involvesthe deliberate exploitation of unpublished price sensitive informationobtained through a privileged relationship to make a profit by dealingin securities of a company when the price of the securities would bealtered if the information were to be disclosed.

Is Susan an insider?

Susan will be an insider if she has information which she knows isinside information and that she knows it is from an inside source:Section 57 CJA 1993. This is a subjective belief and understanding.Anyone can be an insider, but the information must come direct from adirector of the issuer. Susan heard the information during the course ofher employment and from the way in which the question is phrased she isaware that the information is coming from the Chairman of HIT plc.

Is this insider information?

Section 56(1) CJA 1993 describes what constitutes inside information.

It has to relate to particular securities or to a particular issuer of securities.

It is specific or precise. The rumour which Susan has is that thereis to be a takeover bid and this is specific information under thissection. This is not a rumour or casual information.

It has not been made public. Section 58 states publication meansthe information is provided to investors or is to be found in publicrecords. The facts here indicate that this information has not been madepublic.

If made public would be likely to have a significant effect on the price. This condition is satisfied in our present scenario.

The offence in section 52 is dealing with the price-affectedsecurities in a prohibited way. This means dealing with the securitiesyourself, encouraging another to deal in the securities or disclosinginformation other than in the proper course of employment.

In considering whether Susan has encouraged Joseph to deal with thesecurities, it makes no difference whether she stands to make any gainfrom the deal. We are not told what Susan said to James, so it isdifficult to conclude that Susan had encouraged Joseph to deal in theshares. However, in the facts it would appear that Susan will be guiltyof insider dealing as she disclosed the information otherwise than inthe proper performance of her employment and in this situation it isirrelevant whether Susan intends Joseph to act on the information orsuspects that he might.

Defences

Under s53 CJA 1993, Susan will have a defence if she can show that:

(1)She did not expect the dealing in theshares to result in a profit (or the avoidance of a loss) attributableto the fact that the information in question was price sensitive. It isprobably difficult to prove in this situation.

(2)She believed on reasonable grounds that the information had been widely disclosed.

Joseph

Joseph has received the information and knows that it is insideinformation. He will be an insider by virtue of being tipped off bySusan and so will fall under section 57(2)(b) CJA 1993.

He has dealt with securities which are price-affected securities and so has committed an offence under s52(1) CJA 1993.

Defences

The only possible defence is under Sched 1, para 2(1) whichprovides that a person who has market information as an insider has adefence if he can show that it was reasonable for them to have acted asthey did despite being in possession of the information as an insider.

However para 2(2) states account must be taken of the circumstancesand capacity in which the individual first had the information so itlooks as though there will be no defence for Joseph.

Question 28

The stock market value of shares in a company fluctuates inrelation to the underlying performance of the company and theexpectations of investors. Amongst other things, good company resultswill lead to an increase in the value of the shares. Since share pricesfluctuate on the stock market, the possibility arises for individuals tomake large profits, or losses, by speculating in shares. It can alsoprovide people with the opportunity to take advantage of their closerelationship with particular companies in order to make profits fromillegal share dealing. Such illegal trading in shares, known as insiderdealing, occurs when someone trades on the basis of price sensitiveinformation before the general public has access to that information.Insider dealing is governed by Part V of the Criminal Justice Act 1993.

Section 52 of the Criminal Justice Act 1993 states that anindividual, who has information as an insider, is guilty of insiderdealing if he deals in securities that are price-affected securities inrelation to the information. An individual is also guilty of an offenceif he encourages others to deal in securities that are linked with thisinformation, or if he discloses the information otherwise than in theproper performance of his employment, office or profession.

Section 56 makes it clear that securities are price-affected inrelation to inside information if the information, when made public,would be likely to have a significant effect on the price of thosesecurities.

Section 57 defines an insider as a person who knows that he hasinside information and knows that he has the information from an insidesource. This section also states that inside source refers toinformation acquired through being a director, employee or shareholderof an issuer of securities, or having access to information by virtue ofhis employment. Additionally, individuals who acquire their informationfrom those primary insiders (those previously mentioned) are alsoinsiders.

There are a number of defences to a charge of insider dealing. Forexample, s53 makes it clear that no person can be so charged if he didnot expect the dealing to result in any profit or the avoidance of anyloss.

On summary conviction an individual found guilty of insider dealingis liable to a fine not exceeding the statutory maximum and/or amaximum of six months' imprisonment. On indictment the penalty is anunlimited fine and/or a maximum of seven years' imprisonment. There isalso the possibility that the person who benefits from the information,which belongs to the company, will be required to account to the companyfor any profit made. This would certainly be the case with regard todirectors who engaged in insider dealing, as they would have breachedtheir duty to the company.

Applying the law to the situation in the question, it can be seenthat, as an employee of Dome plc, Ewan is an insider under s57, and theinformation he has is certain to affect the price of the company'sshares. It therefore follows that when he buys the shares in Dome plc,Ewan is liable to a charge of insider dealing under s52(1) of theCriminal Justice Act 1993. Ewan is also liable for the separate offence,under s52(2), of disclosing the information to Frank otherwise than inthe proper performance of his employment.

Because he received the information from an insider, Frank istreated as an insider under s57 and is liable for trading on the basisof the information under s52.

Created at 5/24/2012 3:24 PM  by System Account  (GMT) Greenwich Mean Time : Dublin, Edinburgh, Lisbon, London
Last modified at 5/25/2012 12:54 PM  by System Account  (GMT) Greenwich Mean Time : Dublin, Edinburgh, Lisbon, London

Rating :

Ratings & Comments  (Click the stars to rate the page)

Tags:

Recent Discussions

There are no items to show in this view.